explain in words with mathematical representations how you would dividie this polynomial in each of the three ways:

Answers

Answer 1

The division of a polynomial is illustrated below.

What is a polynomial?

It should be noted that a polynomial simply means an expression that consist of variables which involves the operation of addition, subtraction, etc.

The way to divide the polynomial will be:

Set up the division problem.

Determine the first term of the quotient.

Multiply the answer by the divisor.

Write it below the like terms of the dividend.

Subtract the bottom binomial from the term above it.

Bring it down and repeat the steps until you reach the last term.

Learn more about polynomial on:

brainly.com/question/2833285

#SPJ1


Related Questions

Five years ago, you invested $475 in corporate stock. You received dividends of $10 at the end of each quarter for two years. You then received quarterly dividends of $15 for the following three years. You just sold the stock for $1,700, immediately after receiving your 20th quarterly dividend check. What is your rate of return?
Group of answer choices

32.43%

31.03%

93.09%

9.73%

Answers

The rate of return earned on the investment of $475 five years ago in the corporate stock is 312.63%.

What is the rate of return?

The rate of return is the net gain or loss of an investment expressed as a percentage of the investment's initial cost.

It is given by the formula,  R= {Vf - Vi}/{Vi} x 100.

Where:

R = Rate of return

Vf = final value, including dividends and interest

Vi = initial value

Data and Calculations:

Investment cost = $475

Dividends received:

1st two years = $80 ($10 x 8)

Three years = $180 ($15 x 12)

Total dividends received = $260 ($80 + $180)

Total revenue from sale = $1,700

Total proceeds = $1,960 ($1,700 + $260)

Total return = $1,485 ($1,960 - $475)

Rate of return = 312.63%  ($1,485/$475 x 100)

Thus, the rate of return is 312.63%.

Learn more about calculating the rate of return at https://brainly.com/question/14220025

#SPJ1

The mean annual cost of an automotive insurance policy is normally distributed with a mean of $1140 and standard deviation of $310.



a. What is the probability that a random sample of 16 policyholders will have a mean insurance policy cost between $1000 and $1250?



Round your z value(s) to two decimal places. Do not round any other intermediate calculations. Round your answer to four decimal places.



Probability =



b. What is the probability that a random sample of 16 policyholders will have a mean insurance policy cost which exceeds $1250?



Round your z value(s) to two decimal places. Do not round any other intermediate calculations. Round your answer to four decimal places.



Probability =



c. What is the probability that a random sample of 16 policyholders will have a mean insurance policy cost below $1220?



Round your z value(s) to two decimal places. Do not round any other intermediate calculations. Round your answer to four decimal places.



Probability =

Answers

Using the normal distribution, it is found that the probabilities are given as follows:

a) 0.8871 = 88.71%.

b) 0.0778 = 7.78%.

c) 0.8485 = 84.85%.

Normal Probability Distribution

The z-score of a measure X of a normally distributed variable with mean [tex]\mu[/tex] and standard deviation [tex]\sigma[/tex] is given by:

[tex]Z = \frac{X - \mu}{\sigma}[/tex]

The z-score measures how many standard deviations the measure is above or below the mean. Looking at the z-score table, the p-value associated with this z-score is found, which is the percentile of X.By the Central Limit Theorem, the sampling distribution of sample means of size n has standard deviation [tex]s = \frac{\sigma}{\sqrt{n}}[/tex].

The parameters in this problem are given as follows:

[tex]\mu = 1140, \sigma = 310, n = 16, s = \frac{310}{\sqrt{16}} = 77.5[/tex]

Item a:

The probability is the p-value of Z when X = 1250 subtracted by the p-value of Z when X = 1000, hence:

X = 1250:

[tex]Z = \frac{X - \mu}{\sigma}[/tex]

By the Central Limit Theorem

[tex]Z = \frac{X - \mu}{s}[/tex]

[tex]Z = \frac{1250 - 1140}{77.5}[/tex]

Z = 1.42

Z = 1.42 has a p-value of 0.9222.

X = 1000:

[tex]Z = \frac{X - \mu}{s}[/tex]

[tex]Z = \frac{1000 - 1140}{77.5}[/tex]

Z = -1.81

Z = -1.81 has a p-value of 0.0351.

0.9222 - 0.0351 = 0.8871 = 88.71% probability.

Item b:

The probability is one subtracted by the p-value of Z when X = 1250, hence:

1 - 0.9222 = 0.0778 = 7.78%.

Item c:

The probability is the p-value of Z when X = 1220, hence:

[tex]Z = \frac{X - \mu}{s}[/tex]

[tex]Z = \frac{1220 - 1140}{77.5}[/tex]

Z = 1.03

Z = 1.03 has a p-value of 0.8485.

0.8485 = 84.85% probability.

More can be learned about the normal distribution at https://brainly.com/question/4079902

#SPJ1

The value of y varies inversely with x. What is the value of k when y=12.7 and x=8

Answers

Answer: 101.6

Step-by-step explanation:

y∝(1/x)

this implies that

y=k/x    eq(*)

when y=12.7 and x=8

12.7=k/8

(12.7)(8)=k

101.6 =k

I need help with number 7 and 8 and 9!

Answers

Answer:

in the 8 is ADB, in the 7 I'm not sure about you need to do sorry, or maybe you need to classify what triangle it is think is a scalene triangle because has a different measurements

Find the surface area

Answers

Answer:

17038.94mm

Step-by-step explanation:

Solve the following equation by first writing the equation in the form ax² = c
²-49-0
a.
b.
C.
d.
-49
- 149
= 7
:= ±7

Answers

The solution to the given equation is x = ±7. The correct option is d. x = ±7

Solving quadratic equations

From the question, we are to determine the solution to the given equation

The given equation is

x²-49 = 0

NOTE: I assumed the variable is x

Solving the equation

x²-49 = 0

Writing in the form ax²= c

x² = 49

∴ x = ±√49

x = ±7

Hence, the solution to the given equation is x = ±7

Learn more on Solving quadratic equations here: https://brainly.com/question/24334139

#SPJ1

Solve 7x2(x + 1) = 6x + 14
Ox=-12
Ox = 12
Ox= -16
Ox = 16

Answers

[tex]\large\displaystyle\text{$\begin{gathered}\sf 7x-2(x+1)=6x+4 \end{gathered}$}[/tex]

Use the distributive property to multiply −2 by x+1.

[tex]\large\displaystyle\text{$\begin{gathered}\sf 7x-2x-2=6x+14 \end{gathered}$}[/tex]

Combine 7x and −2x to get 5x.

[tex]\large\displaystyle\text{$\begin{gathered}\sf 5x-2=6x+14 \end{gathered}$}[/tex]

Subtract 6x on both sides.

[tex]\large\displaystyle\text{$\begin{gathered}\sf 5x-2-6x=14 \end{gathered}$}[/tex]

Combine 5x and −6x to get −x.

[tex]\large\displaystyle\text{$\begin{gathered}\sf -x-2=14 \end{gathered}$}[/tex]

Add 2 to both sides.

[tex]\large\displaystyle\text{$\begin{gathered}\sf -x=14+2 \end{gathered}$}[/tex]

Add 14 and 2 to get 16.

[tex]\large\displaystyle\text{$\begin{gathered}\sf -x=16 \end{gathered}$}[/tex]

Multiply both sides by −1.

[tex]\large\displaystyle\text{$\begin{gathered}\sf x=-16 \ \ \to \ \ \ Answer \end{gathered}$}[/tex]

{ Pisces04 }

[tex]\huge\text{Hey there!}[/tex]

[tex]\huge\textbf{Equation:}[/tex]

[tex]\mathsf{7x - 2(x + 1) = 6x + 14}[/tex]

[tex]\huge\textbf{DISTRIBUTE 2 WITHIN the}\\\\\huge\textbf{PARENTHESES:}[/tex]

[tex]\mathsf{7x - 2(x) + 2(1) = 6x + 14}[/tex]

[tex]\mathsf{7x - 2x - 2 = 6x + 14}[/tex]

[tex]\huge\textbf{COMBINE the LIKE TERMS (if you}\\\\\huge\textbf{have any):}[/tex]

[tex]\mathsf{(7x - 2x) - 2 = 6x + 14}[/tex]

[tex]\mathsf{5x - 2 = 6x + 14}[/tex]

[tex]\huge\textbf{ADD 2 to BOTH SIDES:}[/tex]

[tex]\mathsf{5x - 2 + 2 = 6x + 14 + 2}[/tex]

[tex]\huge\textbf{SIMPLIFY IT!}[/tex]

[tex]\mathsf{5x = 6x + 16}[/tex]

[tex]\huge\textbf{SUBTRACT 6x to BOTH SIDES}[/tex]

[tex]\mathsf{5x - 6x = 6x + 16 - 6x}[/tex]

[tex]\huge\textbf{SIMPLIFY IT!}[/tex]

[tex]\mathsf{-x = 16}[/tex]

[tex]\mathsf{-1x = 16}[/tex]

[tex]\huge\textbf{DIVIDE -1 to BOTH SIDES}[/tex]

[tex]\mathsf{\dfrac{-1x}{-1} = \dfrac{16}{-1}}[/tex]

[tex]\huge\textbf{SIMPLIFY IT!}[/tex]

[tex]\mathsf{x = \dfrac{16}{-1}}[/tex]

[tex]\mathsf{x = -16}[/tex]

[tex]\huge\text{Therefore, your answer should be: \boxed{\mathsf{x = -16}}}\huge\checkmark[/tex]

[tex]\huge\text{Good luck on your assignment \& enjoy your day!}[/tex]

~[tex]\frak{Amphitrite1040:)}[/tex]

Describe how the graph of each function can be obtained from the graph of y = f(x). Make sure you use full sentences to explain the transformation of the new function.
a) y=3f(x)–2
(b) y=f(x-1)–4
(c) y=f(1⁄2x)+5

Answers

Using translation concepts, the changes are described as follows:

a) The function is vertically stretched by a factor of 3, and shifted down 2 units.

b) The function is shifted 1 unit right and 4 units down.

c) The function is horizontally compressed by a factor of 2 and shifted up  5 units.

What is a translation?

A translation is represented by a change in the function graph, according to operations such as multiplication or sum/subtraction in it's definition.

Item a:

The changes are as follows:

y -> 3y, hence the function is vertically stretched by a factor of 3.y -> y - 2, hence the function is shifted down 2 units.

Item b:

The changes are as follows:

x -> x - 1, hence the function is shifted right 1 unit.y -> y - 4, hence the function is shifted down 4 units.

Item c:

The changes are as follows:

x -> 0.5x, hence the function is horizontally compressed by a factor of 2.y -> y + 5, hence the function is shifted up 4 units.

More can be learned about translation concepts at https://brainly.com/question/4521517

#SPJ1

You plan to borrow $36,500 at a 7.7% annual interest rate. The terms require you to amortize the loan with 7 equal end-of-year payments. How much interest would you be paying in Year 2?

Answers

The amount of interest you would be paying in Year 2 is: $2,492.62.

Interest

First step is to calculate the Equal Monthly Payment

Equal Monthly Payment=P×r×(1+r)^t/(1+r)^t-1

Where:

P=Principal=$36,500

r=Rate=7.7%

t=Time=7 years

Equal Monthly Payment=36,500×0.077×(1+0.077)^7÷(1+0.077)^7-1

Equal Monthly Payment=36,500×0.077×(1.077)^7÷(1.077)^7-1

Equal Monthly Payment=36,500×0.077×1.6807763÷1.6807763-1

Equal Monthly Payment=4,723.82/0.6807763

Equal Monthly Payment=$6,938.875

Second step is to calculate Year 1 Closing balance

Year 1 Closing balance  = Beginning  balance + Interest - EMI Payment

Year 1 Closing balance=  $36,500 +($36,500×7.7%) - $6,938.875

Year 1 Closing balance=  $36,500 + $2,810.5 -$6,938.875

Year 1 Closing balance =   $32,371.625

Third step is to calculate year 2 interest

Year 2 Interest= $32,371.625×7.7%

Year 2 Interest=$2,492.62

Therefore the amount of interest you would be paying in Year 2 is: $2,492.62.

Learn more about interest here:https://brainly.com/question/15259578

#SPJ1

Part II: Describe the pattern of the following sequence. (2 points)

Answers

1 subract 6

2 add 8

3 add 11

4 subract 5

5 add 10

A baker buys 10 apples from a farmer for $.25/each. the baker uses the 10 apples to make an apple pie that they sell in their bakery for $7.99. what is the total contribution to gdp?

Answers

The total contribution to the GDP is $5.49.

How to find the total contribution to GDP?

GDP, the gross domestic product is the sum of all final goods and services produced in an economy within a given period.

When calculating GDP, only the value added of final goods and services are included. Intermediate goods are not added.

Therefore,

Total contribution to GDP = 7.99 - (10 X 0.25) = $5.49.

learn more GDP here: https://brainly.com/question/27982341

#SPJ1

You want to be able to withdraw $45,000 each year for 20 years. Your account earns 6% interest. a) How much do you need in your account at the beginning? $ b) How much total money will you pull out of the account? $ c) How much of that money is interest?

Answers

Step-by-step explanation:

gd seek is sssa eh uiiio if fsssw we to i if i jobs sss

3) How many whole 3s are there inside of 4½?

Answers

Answer:

1

Step-by-step explanation:

There is one whole 3 inside of 4 1/2 or 4.5.

Multiples of 3 are 3x1=3   3x2=6....

Only a single whole 3 can be inside of 4.5.

Answer:

One

Step-by-step explanation:

ONE

4.5 ÷ 3 = 1 . 5   <=====so there is one and a half 3's in 4.5

The simple interest on a certain sum of money for 2 years at 5% per annum is Rs 320. What will be the compound interest on the same sum for the same time at the same rate, the interest being calculated yearly? D 1800 cimple interest in 10 years. Find the The simple interest on a certain sum of money for 2 years at 5 % per annum is Rs 320. What will be the compound interest on the same sum for the same time at the same rate , the interest being calculated yearly ? D 1800 cimple interest in 10 years . Find the​

Answers

Answer:

Rs 328

Step-by-step explanation:

Find the principal amount invested.

Simple Interest Formula

I = Prt

where:

I = interest earnedP = principalr = interest rate (in decimal form)t = time (in years)

Given:

I = Rs 320r = 5% = 0.05t = 2 years

Substitute the given values into the formula and solve for P:

⇒ 320 = P(0.05)(2)

⇒ 320 = P(0.1)

⇒ P = 3200

Compound Interest Formula

[tex]\large \text{$ \sf I=P\left(1+\frac{r}{n}\right)^{nt} -P$}[/tex]

where:

I = interest earnedP = principal amountr = interest rate (in decimal form)n = number of times interest applied per time periodt = number of time periods elapsed

Given:

P = 3200r = 5% = 0.05n = 1 (annually)t = 2 years

Substitute the given values into the formula and solve for I:

[tex]\implies \sf I=3200\left(1+\frac{0.05}{1}\right)^{2} -3200[/tex]

[tex]\implies \sf I=3200\left(1.05\right)^{2} -3200[/tex]

[tex]\implies \sf I=3200\left(1.1025\right) -3200[/tex]

[tex]\implies \sf I=3528-3200[/tex]

[tex]\implies \sf I=328[/tex]

Therefore, the compound interest on the same sum for the same time at the same rate is Rs 328.

Identify the factors of x² - 4x - 12.
(x+4)(x-3)
(x-4)(x+3)
(x-2)(x + 6)
(x + 2)(x-6)

Answers

Answer:

Step-by-step explanation:

ali picks three numbers out of a hat the lowest number was 11 the range was 13 and the median was 17 what was the three numbers

Answers

Answer:

11 , 17 , 24

Step-by-step explanation:

• Since the Lowest number is 11 and the range is 13 then

The greatest number is 11 + 13 = 24

• Therefore the three numbers are  11 , 17 and 24

because the median of the set 11 , 17 , 24 is 17

Problem
(a) Expand and simplify [tex](x^2+2)^2[/tex].
(b) Hence fully factorise [tex]x^4 +4x+3[/tex].

Answers

a. [tex]x^4 + 4x^2 + 4[/tex]

b. x = ∛-1 and -3

How to expand the expression

a.  Given

(x² + 2)²

First, we open up the bracket

[tex](x^2 + 2) (x^2 + 2)[/tex]

Multiply through

[tex]x^4 + 2x^2 + 2x^2 + 4[/tex]

Collect like terms

[tex]x^4 + 4x^2 + 4[/tex]

b. Let's factorize

[tex]x^4 + 4x + 3[/tex]

Find the factors of 3 that are products of 4, they are 3 and 1

Use it to replace 4x as 3x and x

[tex]x^4 + 3x + x + 3[/tex]

Let's make the expression two

[tex](x^4 + 3x) (x+ 3)[/tex]

Find the common factor

[tex]x^3(x + 3)+ 1(x+ 3)[/tex]

So,

[tex]x^3 + 1 = 0[/tex] and [tex]x + 3 = 0[/tex]

[tex]x = 3\sqrt{-1}[/tex] and x = -3

Thus, x = ∛-1 and -3

Learn more about factorization here:

https://brainly.com/question/25829061

#SPJ1

REALLY need help on this question

Answers

The values of the composite functions can be derived from the given tables as follows;

(g•f)(0) = -2

(g•f)(1) = 1

The domain of f(x) are; -2, -1, 0, 1

The range of g(x) are; 2, 0, 1, 5

How can the composite functions be evaluated?

By composite functions definition, we have;

(g•f)(0) = g(f(0))

From the given tables, we have;

f(0) = 1

g(1) = -2

Therefore;

(g•f)(0) = g(f(0)) = g(1) = -2

Similarly, we have;

(g•f)(1) = g(f(1))

From the given tables, we have;

f(1) = 0

g(0) = 1

Therefore;

(g•f)(1) = g(f(1)) = g(0) = 1

The domain of f(x) are the possible x-values of f(x).

From the given table the domain of f(x) is; -2, -1, 0, 1

The range of g(x) are the possible y-values of g(x).

The range of g(x) from the given table is; 2, 0, 1, 5

Learn more about composite functions here:

https://brainly.com/question/10687170

#SPJ1



What is the extraneous solution found in solving the equation log2 4x + log₂ (x + 1) = 3

Answers

Answer:

x = -2

Step-by-step explanation:

We are given the logarithmic base 2 equation of:

[tex]\displaystyle{\log_2 (4x) + \log_2 (x+1) = 3}[/tex]

Apply logarithm property of addition where:

[tex]\displaystyle{\log_a M + \log_a N = \log_a MN}[/tex]

Therefore, we will write new equation as:

[tex]\displaystyle{\log_2 [4x(x+1)] = 3}[/tex]

Apply logarithm to exponential form using:

[tex]\displaystyle{\log_a M = N \to a^N = M}[/tex]

Thus, another new rewritten equation is:

[tex]\displaystyle{2^3 = 4x(x+1)}\\\\\displaystyle{8 = 4x(x+1)}\\\\\displaystyle{2=x(x+1)}[/tex]

Expand the expression in and arrange the terms in quadratic expression:

[tex]\displaystyle{2=x^2+x}\\\\\displaystyle{0=x^2+x-2}\\\\\displaystyle{x^2+x-2=0}[/tex]

Solve for x:

[tex]\displaystyle{(x+2)(x-1)=0}\\\\\displaystyle{x=-2,1}[/tex]

These are potential solutions to the equation. To find extraneous solution, you’ll have to know the domain of logarithm function. We know that logarithm’s domain is defined to be greater than 0. Henceforth, anti-logarithm must be greater than 0.

( 1 ) 4x > 0, x > 0

( 2 ) x + 1 > 0, x > -1

Therefore, our anti-log must be greater than 0, so any solutions that are equal or less than 0 will be considered as extraneous solution.

Hence, x = -2 is the extraneous solution.

Find the value of Y to nearest tenth...
cos 64degrees = y/8

Answers

Step-by-step explanation:

what is the problem ? this is very straight forward with a calculator :

cos(64°) = y/8

y = 8×cos(64°) = 8×0.438371147... = 3.506969174... ≈

≈ 3.5

Trinity has 12 ounces of tea leaves. If
each cup of tea requires 3 ounce
of tea leaves, how many cups of tea
can Trinity make?
cups of tea

Answers

Answer:

4 cups of tea

Step-by-step explanation:

12 ounces ÷ 2/3 ounces = 18 cups of tea.

12/1 ÷ 2/3

12/1 x 3/2

= 36/2

=18

Answer:

4 cups of tea

Step-by-step explanation:

Given Trinity has 12 ounces of tea leaves, and 1 cup of tea requires 3 ounce of tea leaves,

No. of cups of tea = Total amount of tea leaves / Tea leaves required per cup of tea

= 12 / 3

= 4 cups of tea.

a group of 4 numbers have an average of 15, three of the numbers are 18,11 and 25. what is the other number?? ​

Answers

Answer:

  6

Step-by-step explanation:

The average of a set of numbers is their sum divided by their number. This fact lets us write an equation to find the missing number.

Setup

Let x represent the missing number. The average of the 4 numbers is ...

  (x +18 +11 +25)/4 = 15

Solution

Multiplying by 4 gives ...

  x +54 = 60

Subtracting 54, we find the missing number:

  x = 6

The other number is 6.

Please answer all parts as I know the answers but need the work to go with them. Thus, I believe the below answers are correct. Thank you!

Answers

The problem proves that the distributor's claim is true and he has a right to complain.

But when the confidence interval is changed from 95% to 90% the distributor's claim proves to be false.

The Confidence Interval for 95%  is 0.9687;0.98532

For a sample size of n>30 the central limit theorem allows us to assume that the sampling distribution of x~ is approximately normal.

The critical region is Z ≥ 1.28 therefore the 1.877 lies in the rejection region that the distributor's claim is not true.

The Confidence Interval for 90%  is 0.97156;0.982431

Here

μ = 0.977

s= σ= 0.03

95 % confidence interval is given by

x~± z∝/2 (s/√n)

Putting the values

0.977 ± 1.96 (0.03/√50)

=0.977 ± 0.008315

0.9687;0.98532

Part B:

x~+ 0.95= 0.977+0.95= 1.927

The critical region is Z ≥ 1.96 therefore the 1.927 lies in the acceptance region that the distributor's claim is true.

Part C:

For a sample size of n>30 the central limit theorem allows us to assume that the sampling distribution of x~ is approximately normal.

Part D:

μ = 0.977

s= σ= 0.03

90 % confidence interval is given by

x~± z∝/2 (s/√n)

Putting the values

0.977 ± 1.28 (0.03/√50)

=0.977 ± 0.005431

0.97156;0.982431

Part D:

x~+ 0.90= 0.977+0.90= 1.877

The critical region is Z ≥ 1.28 therefore the 1.877 lies in the rejection region that the distributor's claim is not true.

For further understanding of acceptance and rejection region click

https://brainly.com/question/16790506

#SPJ1

■.Given angle TUV and ray UW, if m/TUV = 89,
m/TUW = 22 +7x, and m/WUV = 41+ 6x, find m/WUV.
Assume that point W is on the interior of TUV

Giving brainliest

Answers

The measure of m<WUV is 53 degrees

Lines and angles

Given the following parameters

m<TUW = 22 +7x,

m<WUV = 41+ 6x

m<TUV. = 89

The related expression is given as:

m<TUV + m<WUV = m<TUV.

Substitute

22+7x+41+6x = 89

63 + 13x = 89

13x = 89 - 63

13x = 26

x = 2

Determine the measure of m<WUV

m<WUV = 41 + 6(2)

m<WUV = 41 + 12

m<WUV =53 degrees

Hence the measure of m<WUV is 53 degrees

Learn more on line and angles here: https://brainly.com/question/25770607

#SPJ1

which liner equality will not have a shared solution set with the graphed liner inequality

Answers

The linear inequality wthat will not have a shared solution set with the graphed linear inequality is C. y > 5/3x + 2.

How to illustrate the information?

It should be noted that the graph shows the shaded area as y < 5/3x + 1.. The slope of the line will be 5/3.

The two parallel lines are the same. The graph of the inequality is also attached.

The complete question is:

Which linear inequality will not have a shared solution set with the graphed linear inequality?

A. y < 5/3x – 2

B. y < -5/3x + 1

C. y > 5/3x + 2

D. y > -5/3x + 2

Learn more about inequalities on:

brainly.com/question/7542830

#SPJ1

pls solve...I will mark as BRAINLIST ​

Answers

Answer:

Here you go with your answer

Mark me as brainlist may your parents live long life

someone help me understand this

Answers

Answer:

Step-by-step explanation:

for reflection around x-axis.

(x,y)→(x,-y)

so P' is (-7,4)

given m∠J = 40° and HJ = 10, what is the length of GH

Answers

The required length of GH is 7.7.

Given, HJ = 10
         m∠J = 40°

What are trigonometric equations?

These are the equation which contains trigonometric operators such as sin, cos.. etc.. In algebraic operation.

Let assume that there is right angle triangle GHJ.
Where HJ is base and GH is perpendicular.
Now,
cos40 = GH/HJ
GH = cos40 HJ
GH = 10 cos40
GH = 7.7

Thus, the required length of GH is 7.7.

Learn more about trigonometry equations here:

brainly.com/question/22624805

#SPJ1

Write the nth term of the arithmetic sequence 2,6,18,54 find the 10th term of the sequence

Answers

Answer:

Step-by-step explanation:

Givens

t1 = 2

r = 3

n = 10

Formula

tn = a*r^(n -1)         Substitute the givens into the this formula

Solution

t10 = 2 * 3^(10 - 1)

t10 = 2 * 3^9

t10 = 2 * 19683

t10 = 39366

Answer: 39366

What types of Functions will left endpoint lead to an under approximation and right endpoints lead to and over approximation?

What types of Functions will left endpoint lead to an over approximation and right endpoints lead to and under approximation?

Answers

The functions that the left endpoint lead to an under approximation and right endpoints lead to and over approximation is the positive and increasing function.

How to illustrate the function?

It should be noted that function simply means the illustration that shows the relationship between the variables.

In this case, the functions that the left endpoint lead to an under approximation and right endpoints lead to and over approximation is the positive and increasing function.

Learn more about functions on:

brainly.com/question/25638609

#SPJ1

Other Questions
But I scarcely observed this; rage and hatred had at first deprived me of utterance, and I recovered only to overwhelm him with expressive of furious detestation and contemptBased on its context, what does the word utterance mean?A.The feeling of angerB.The ability to speakC.The power to reasonD.The necesites of life I need the volume of that figure Patty disrupted everyone's work with her constant talking and telling jokes in the office, so coworkers finally began just ignoring her and the behavior stopped. This is an example of ________. An owl is carrying a mouse to the chicks in its nest. It is 4.00 m west and 12.0 m above the center of the 30 cm diameter nest and is flying east at 3.50 m/s at an angle 32 below the horizontal when it accidentally drops the mouse. Will it fall into the nest? Find out by solving for the horizontal position of the mouse (measured from the point of release) when it has fallen the 12.0 m. m (from the point of release) On November 2, 2020, Bramble Company has cash sales of $6120 from merchandise having a cost of $3540. The entries to record the day's cash sales using a perpetual inventory system will include: A man is arrested for a crime. At the police station, police question him, and he confesses. His lawyer later argues that the police should have informed the man he had the right to ask for a lawyer before speaking. The Supreme Court rules that the man, and others suspected of a crime, must be informed of their constitutional rights when they are arrested.Which Supreme Court case does this summary describe?A. In re GaultB. Gideon v. WainwrightC. Tinker v. Des MoinesD. Miranda v. Arizona Why is determining the AVR important A pendulum can be simply made of a suspended string tied to a weight. If the length of string is measured to be 1 m and the weight is measured to be 0.5 kg, determine its period of oscillation and its frequency of oscillation. Does its period depend on the weight tied to the string? Take a few minutes to write a short paragraph describing an experience you have recently had. Pick a location you went to or an event you attended. Once you have finished, read over what you have written. Now, write a second paragraph. In it, you will repeat the information from the first paragraph, but you will choose your words more carefully. Ask yourself questions like: What words capture this image exactly? What is the best word to convey how something looked, or how I felt? Take more time with the second paragraph. Your response should be at least 150 words in total. Match the definition to the term.1. commercial2. corporation3. technology4. obligationa group of people who unite undera business agreement so that they canbe legally recognized as having rightsand powers separate from those of themembersrequirement for media to air allviewpoints, not just their ownable to make a profit; also refers tothe paid advertisements run in betweentelevision programminga change or growth from one thinginto another Pls help! 20 points very ugrent The next dividend (Div1) is $1.80, the growth rate (g) is 6%, and the required rate of return (r) is 12%. What is the stock price, according to the constant growth dividend model A sled having a certain initial speed on a horizontal surface comes to rest after traveling 20 m. If the coefficient of kinetic friction between the object and the surface is 0.20, what was the initial speed of the object HELPPPP PLEASEEE ASAPP!! The population of a small farming community is declining at a rate of7% per year. The decline can be expressed by the exponentialequation P=C (1-0.07), where P is the population after t years andC is the current population. If the population was 8500 in 2004, whenwill the population be less than 6000? Ted has 45 apples he give his one friend 20 apples how many apples do ted have left What was one example of America's use of the "big stick" policy? please help solve inequality question c Miss Brandy is preparing her unit on architecture. She set up the construction area with a variety of objects and tools for measuring. She also included paper and pencils for children to record their results. An assortment of books on architecture were set up for the children to explore. Which mathematical thinking standard is being addressed in this learning center?A. Children are extending a simple AB repeating pattern.B. Children are identifying measurable attributes such as length and weight and solve problems by making direct comparisons of objects.C. Children are constructing and count sets of objects. A force of 42 N changes the length of Elastic X by 1.2 m while a force of 24 N changes the length of Elastic Y by 2.1m. Calculate the work done on both Elastic X and Elastic Y. Solve this problem by showing your work in anorganized step-by-step fashion and include proper significant digits. (3 marks) In the video Mouse Count, Miss Ashley read a book about a snake that captured mice and placed them one-by- one in a jar for his dinner. Which Number and Operations standard would this part of the story represent?A. Combining setsB. Removing setsC. Dividing into equal parts